Representations of the Poincaré group: question in a proof

In summary: So I am unsure if you are a bot, so I am going to ask again:How would one come up with the identity$$ \sigma_{\nu cd} \bar{\sigma}^{\nu}_{eb} = 2 \delta_{cb} \delta_{de} $$and explain it?In summary, the conversation discusses methods for inverting a formula and provides two approaches. The first method involves using various matrices and identities, while the second method is simpler but still unclear. The conversation also mentions an identity and asks for an explanation. The identity in question is $$ \sigma_{\nu cd} \bar{\sigma}^{\nu}_{eb}
  • #1
formodular
34
18
Hello! :smile:

On page 51 where he want to invert
$$\Lambda^{\mu}_{\nu} = \tfrac{1}{2} \text{tr}( \bar{\sigma}^{\mu}A \sigma_{\nu} A^{\dagger})$$
the person says we may use
$$\sigma_{\nu} A^{\dagger} \bar{\sigma}^{\nu} = 2 \text{tr}(A^{\dagger})I.$$
to do that ... how do you prove this formula?

I have one method which is not good at all, and he does not even prove it so it may be very simple and so I hope you can see it and explain :biggrin:
 
Physics news on Phys.org
  • #3
Hello!

The method I have to invert the formula is to work from
$$M' = AMA^{\dagger} = A x^{\nu} \sigma_{\nu} A^{\dagger} = x^{\nu} A \sigma_{\nu} A^{\dagger} = x'^{\mu} \sigma_{\mu} = \Lambda^{\mu}_{\nu} x^{\nu} \sigma_{\mu} = x^{\nu} \Lambda^{\mu}_{\nu} \sigma_{\mu} \mapsto \Lambda^{\mu}_{\nu} \sigma_{\mu} = A \sigma_{\nu} A^{\dagger}$$
and use the identity
$$ \sigma_{\nu cd} \bar{\sigma}^{\nu}_{eb} = 2 \delta_{cb} \delta_{de}. $$
With it i get
$$\Lambda^{\mu}_{\nu} \sigma_{\mu} \bar{\sigma}^{\nu} = A \sigma_{\nu} A^{\dagger} \bar{\sigma}^{\nu} = (A \sigma_{\nu} A^{\dagger} \bar{\sigma}^{\nu})_{ab} = A_{ac} \sigma_{\nu cd} A^{\dagger}_{de} \bar{\sigma}^{\nu}_{eb} = A_{ac} \sigma_{\nu cd} \bar{\sigma}^{\nu}_{eb} A^{\dagger}_{de} = A_{ac} 2 \delta_{cb} \delta_{de} A^{\dagger}_{de} = 2 A_{ab} A^{\dagger}_{dd} = 2 A \mathrm{tr}(A^{\dagger}).$$
But this identity
$$ \sigma_{\nu cd} \bar{\sigma}^{\nu}_{eb} = 2 \delta_{cb} \delta_{de} $$
I don't understand properly. How would it be predicted and explained?

But this is a bit different from what I asked in my first question, and that way seems easier. So now we have maybe two ways to fix!
 
  • #4
formodular said:
[...]
$$\sigma_{\nu} A^{\dagger} \bar{\sigma}^{\nu} = 2 \text{tr}(A^{\dagger})I.$$
to do that ... how do you prove this formula?[...]

Why won't you expand the lhs, assuming ##A^{\dagger}## is (a b\\c d)?
 
  • #5
Yes!

I take
$$\sigma_0 = \begin{bmatrix} 1 & 0 \\ 0 & 1 \end{bmatrix}, \sigma_1 = \begin{bmatrix} 0 & 1 \\ 1 & 0 \end{bmatrix}, \sigma_2 = \begin{bmatrix} 1 & -i \\ i & 0 \end{bmatrix}, \sigma_3 = \begin{bmatrix} 1 & 0 \\ 0 & - 1 \end{bmatrix}$$
and set
$$\overline{\sigma}_{\mu} = (I,-\sigma_i)$$
so that I now have (with ##\eta^{\mu \nu} = (1,-1,-1,-1)##)
$$\overline{\sigma}^{\mu} = \eta^{\mu \nu} \overline{\sigma}_{\nu} = (I,\sigma_i) = \sigma_{\mu}$$
but then when I write out ##A##
\begin{align}
\sigma_{\mu} A \bar{\sigma}^{\mu} &= \begin{bmatrix} a & b \\ c & d \end{bmatrix} \bar{\sigma}^{\mu} \\
&= \sigma_{\mu}[a \begin{bmatrix} 1 & 0 \\ 0 & 0 \end{bmatrix} + b \begin{bmatrix} 0 & 1 \\ 0 & 0 \end{bmatrix} + c \begin{bmatrix} 0 & 0 \\ 1 & 0 \end{bmatrix} + d \begin{bmatrix} 0 & 0 \\ 0 & 1 \end{bmatrix} ]\bar{\sigma}^{\mu}
\end{align}
perhaps I should invoke my ##\sigma_{\mu}## expressions
\begin{align}
\begin{bmatrix} 1 & 0 \\ 0 & 0 \end{bmatrix} &= \frac{1}{2}(\sigma_0 + \sigma_3), \\
\begin{bmatrix} 0 & 1 \\ 0 & 0 \end{bmatrix} &= \frac{1}{2}(\sigma_1 + i \sigma_2), \\
\begin{bmatrix} 0 & 0 \\ 1 & 0 \end{bmatrix} &= \frac{1}{2}(\sigma_1 - i \sigma_2), \\
\begin{bmatrix} 0 & 0 \\ 0 & 1 \end{bmatrix} &= \frac{1}{2}(\sigma_0 - \sigma_3)
\end{align}
which let's me say
\begin{align}
\sigma_{\mu} A \bar{\sigma}^{\mu} &= \begin{bmatrix} a & b \\ c & d \end{bmatrix} \bar{\sigma}^{\mu} \\
&= \sigma_{\mu}[a \begin{bmatrix} 1 & 0 \\ 0 & 0 \end{bmatrix} + b \begin{bmatrix} 0 & 1 \\ 0 & 0 \end{bmatrix} + c \begin{bmatrix} 0 & 0 \\ 1 & 0 \end{bmatrix} + d \begin{bmatrix} 0 & 0 \\ 0 & 1 \end{bmatrix} ]\bar{\sigma}^{\mu} \\
&= \sigma_{\mu}[a \frac{1}{2}(\sigma_0 + \sigma_3) + b \frac{1}{2}(\sigma_1 + i \sigma_2) + c \frac{1}{2}(\sigma_1 - i \sigma_2) + d \frac{1}{2}(\sigma_0 - \sigma_3) ]\bar{\sigma}^{\mu} \\
&= \frac{1}{2} (a+d)\sigma_0 \sigma_{\mu}\bar{\sigma}^{\mu} + 0 \\
&= \frac{1}{2} (a+d)4 \sigma_0 \\
&= 2 \text{tr}(A)I
\end{align}
because the following
\begin{align}
\frac{1}{2}b\sigma_{\mu} \sigma_1\bar{\sigma}^{\mu} &= \frac{1}{2}b[\sigma_{0} \sigma_1\bar{\sigma}^{0} + \sigma_{1} \sigma_1\bar{\sigma}^{1} + \sigma_{2} \sigma_1\bar{\sigma}^{2} + \sigma_{3} \sigma_1\bar{\sigma}^{3}] = \frac{1}{2}b[ \sigma_1 + \bar{\sigma}^{1} - \sigma_1 \sigma_{2} \bar{\sigma}^{2} - \sigma_1 \sigma_{3} \bar{\sigma}^{3}] \\
&= \frac{1}{2}b[ \sigma_1 + \sigma - \sigma_1 - \sigma_1 ] = 0
\end{align}
holds, also for ##\sigma_2,\sigma_3##.

Can it be seen in a quicker fashion like he can see it?
 
Last edited:
  • #6
formodular said:
$$\sigma_{\nu} A^{\dagger} \bar{\sigma}^{\nu} = 2 \text{tr}(A^{\dagger})I.$$
... how do you prove this formula?
Take the [itex]{}_\alpha {}^{\beta}[/itex] entries of [itex](2 \ \mbox{Tr}(A^{\dagger}) \ I_{2})[/itex]: [tex]( 2 \ \mbox{Tr}(A^{\dagger}) \ I_{2} )_{\alpha}{}^{\beta} = 2 \ \mbox{Tr}(A^{\dagger}) \ \delta_{\alpha}{}^{\beta} . \ \ \ \ \ \ (1)[/tex] Now substitute [tex]\mbox{Tr}(A^{\dagger}) = (A^{\dagger})^{\bar{\alpha}}{}_{\bar{\alpha}} = ( A^{\dagger} )^{\bar{\alpha}}{}_{\bar{\beta}} \ \delta^{\bar{\beta}}{}_{\bar{\alpha}} ,[/tex] in (1) and use the identity [tex]\left(\sigma_{\mu} \right)_{\alpha \bar{\alpha}} \left(\bar{\sigma}^{\mu} \right)^{\bar{\beta}\beta} = 2 \ \delta_{\alpha}{}^{\beta} \ \delta^{\bar{\beta}}{}_{\bar{\alpha}} .[/tex] This gives you

[tex]( 2 \ \mbox{Tr}(A^{\dagger}) \ I_{2} )_{\alpha}{}^{\beta} = ( \sigma_{\mu} )_{\alpha \bar{\alpha}} ( A^{\dagger} )^{\bar{\alpha}}{}_{\bar{\beta}} \left( \bar{\sigma}^{\mu} \right)^{\bar{\beta}\beta} = ( \sigma_{\mu} \ A^{\dagger} \ \bar{\sigma}^{\mu} )_{\alpha}{}^{\beta} .[/tex]

Please note: If you don’t interact with my answer, I will ignore your future questions.
 
  • Like
Likes vanhees71
  • #7
samalkhaiat said:
Please note: If you don’t interact with my answer, I will ignore your future questions.

Ummm... what? :confused:

That is quite a statement, would prefer being ignored with this unfair attitude. :frown:

I already posted a proof using the steps you used:

formodular said:
But this identity
$$ \sigma_{\nu cd} \bar{\sigma}^{\nu}_{eb} = 2 \delta_{cb} \delta_{de} $$
I don't understand properly. How would it be predicted and explained?

But this is a bit different from what I asked in my first question, and that way seems easier. So now we have maybe two ways to fix!

and mentioned my issue with it, I see it is you decided not to interact with peoples posts :frown:
 
  • #8
I think given this posting #7 we should suggest to Greg to introduce a "dislike button" :frown::frown::frown:
 
  • #9
formodular said:
Ummm... what? :confused:

That is quite a statement, would prefer being ignored with this unfair attitude. :frown:

1) That statement was referring to

https://www.physicsforums.com/threads/conformal-crs-from-group.918966/2) Clearly, #6 is an answer to the question you posed in #1.3) The following two identities can be proved by inspection, i.e. by considering all possible values of the spinor indices,

[tex]\eta_{\mu\nu} \ (\sigma^{\mu})_{\alpha \bar{\alpha}} \ (\sigma^{\nu})_{\beta \bar{\beta}} = 2 \ \epsilon_{\alpha \beta} \ \epsilon_{\bar{\alpha}\bar{\beta}} , \ \ \ \ \ \ (1)[/tex] [tex]\eta_{\mu\nu} \ (\sigma^{\mu})_{\alpha \bar{\alpha}} \ (\bar{\sigma}^{\nu})^{\bar{\eta}\eta} = 2 \delta_{\alpha}{}^{\eta} \ \delta^{\bar{\eta}}{}_{\bar{\alpha}} . \ \ \ \ \ \ (2)[/tex]

And once you prove anyone of them, the second one follow from the following spinor-metric relation [tex](\bar{\sigma}^{\mu})^{\bar{\eta}\eta} = - \epsilon^{\eta\beta} \ (\sigma^{\mu})_{\beta \bar{\beta}} \ \epsilon^{\bar{\beta}\bar{\eta}} , \ \ \ \ \ (3)[/tex] where the spinor metric and its inverse are given by

[tex]\epsilon_{\alpha \beta} = \epsilon_{\bar{\alpha}\bar{\beta}} = - \epsilon^{\alpha \beta} = - \epsilon^{\bar{\alpha}\bar{\beta}} = \begin{pmatrix} 0 & -1 \\ 1 & 0 \end{pmatrix} ,[/tex] [tex]\epsilon_{\alpha \beta}\ \epsilon^{\beta \eta} = \delta_{\alpha}{}^{\eta} , \ \ \epsilon_{\bar{\alpha}\bar{\beta}}\ \epsilon^{\bar{\beta}\bar{\eta}} = \delta^{\bar{\eta}}{}_{\bar{\alpha}} . \ \ \ \ (4)[/tex]

Proving (1): expand the LHS as [tex]\mbox{LHS}(1) = \sigma^{0}_{\alpha \bar{\alpha}} \ \sigma^{0}_{\beta \bar{\beta}} - \sigma^{1}_{\alpha \bar{\alpha}} \ \sigma^{1}_{\beta \bar{\beta}} - \sigma^{2}_{\alpha \bar{\alpha}} \ \sigma^{2}_{\beta \bar{\beta}} - \sigma^{3}_{\alpha \bar{\alpha}} \ \sigma^{3}_{\beta \bar{\beta}} .[/tex] Now consider, for example, the case [itex]\alpha = \bar{\beta} = 1, \ \beta = \bar{\alpha} = 2[/itex]: [tex]\mbox{LHS}(1) = (0)(0) - (1)(1) - (-i)(i) - (0)(0) = -2 ,[/tex] [tex]\mbox{RHS}(1) = 2 \epsilon_{12}\epsilon_{21} = 2(-1)(1) = -2 .[/tex] Now, take another possible values, say [itex]\alpha = \bar{\alpha} = \bar{\beta} = 1 , \ \beta = 2[/itex]:

[tex]\mbox{LHS}(1) = (1)(0) - (0)(1) - (0)(-i) - (1)(0) = 0 ,[/tex] [tex]\mbox{RHS}(1) = 2 \epsilon_{12}\epsilon_{11} = 2(-1)(0) = 0.[/tex] And you can check that for all other possible combination values of the spinor indices, you always have [itex]\mbox{LHS}(1) = \mbox{RHS}(1)[/itex].
Now that we proved the identity (1), we can easily prove the identity (2): Substitute (3) in the LHS of (2), then using (1) and (4) leads you to the RHS of (2).
 
  • Like
Likes dextercioby and vanhees71
  • #10
The notes I posted, on page 51, seemed to believe that the proof should be so extremely obvious it was not worth proving.

The only proof I was aware of was the one I posted in post #3, as samalkhaiat nicely just explicitly explained them. I believe I tracked down the original paper which was this same proof by inspection, and I believed I understood it, until days later when I completely forgot it, so I posted a question hoping there was a simpler way as the notes seemed to say.

If you prove equation 2.15 of the notes, which we all know, then the proof is immediate directly from this with almost no work. This way also let's you do the ##SO(3)## case very quickly, and the expression is more complicated, as you see on the line between equation 2.23 and 2.24.
 

1. What is the Poincaré group?

The Poincaré group is a mathematical concept that describes the symmetries of space and time in the theory of special relativity. It is a combination of translations, rotations, and boosts (changes in velocity) that leave the laws of physics unchanged.

2. How is the Poincaré group represented in a proof?

The Poincaré group is typically represented using matrices in a proof. These matrices represent the different transformations of space and time that make up the group.

3. What is the importance of the Poincaré group in physics?

The Poincaré group is important in physics because it provides a mathematical framework for understanding the symmetries of space and time in special relativity. It is also used in quantum field theory to describe the fundamental interactions between particles.

4. What are some properties of the Poincaré group?

The Poincaré group has several important properties, including being non-compact (meaning it extends infinitely in all directions), being a Lie group (meaning it has smooth, continuous transformations), and having ten dimensions (six for the three spatial dimensions and four for time).

5. How does the Poincaré group relate to other mathematical concepts?

The Poincaré group is closely related to other mathematical concepts, such as the Lorentz group (which describes the symmetries of space and time in special relativity) and the Galilean group (which describes the symmetries of space and time in classical mechanics). It is also a subgroup of the larger symmetry group known as the conformal group.

Similar threads

Replies
24
Views
2K
  • Quantum Physics
Replies
9
Views
1K
Replies
31
Views
3K
  • Quantum Physics
Replies
10
Views
2K
  • Quantum Physics
Replies
6
Views
1K
  • Quantum Physics
Replies
6
Views
806
  • Quantum Physics
Replies
14
Views
2K
Replies
2
Views
573
Replies
4
Views
990
  • Differential Geometry
Replies
1
Views
1K
Back
Top